Medsurg NCLEX Questions - Comprehensive Respiratory

¡Supera tus tareas y exámenes ahora con Quizwiz!

Guaifenesin 300 mg four times daily has been ordered as an expectorant. The dosage strength of the liquid is 200mg/5ml. How many mL should the nurse administer each dose? Fill in the blank and record your final answer using one decimal place.

7.5 mL When the medicine is a solution of specific strength, calculations can become more complicated. Liquids (solutions and suspensions) are frequently used in children's nursing - for example for children who find swallowing tablets difficult or patients who have medicines administered via a percutaneous endoscopic gastrostomy (PEG) tube.

The nurse is teaching the client how to use a metered-dose inhaler (MDI) to administer a Corticosteroid drug. Which of the following client actions indicates that he is using the MDI correctly? Select all that apply. A. The inhaler is held upright. B. Head is tilted down while inhaling the medication. C. Client waits 5 minutes between puffs. D. Mouth is rinsed with water following administration. E. Client lies supine for 15 minutes following administration.

A and D Inhaled respiratory medications are often taken by using a device called a metered-dose inhaler, or MDI. The MDI is a pressurized canister of medicine in a plastic holder with a mouthpiece. When sprayed, it gives a reliable, consistent dose of medication. Option A: Remove the cap and hold the inhaler upright. Each inhaler consists of a small canister of medicine connected to a mouthpiece. The canister is pressurized. As the client presses down on the inhaler, it releases a mist of medicine. The client breathes that mist into the lungs. It's important to use the inhaler correctly. Option B: Tilt the head back slightly and breathe out all the way. Keep the chin up and the inhaler upright (not aimed at the roof of the mouth or the tongue). Option C: Repeat puffs as directed by the doctor. Wait 1 minute before taking the second puff. A delay of 10-20 minutes between successive doses of the bronchodilator drug has been suggested in order to let the first act to improve the penetration and effect of the second dose, but again the evidence that this works is inconclusive. Many patients may forget to take a second dose with such a long interval. Option D: Some inhalers (steroid) also recommend rinsing the mouth out with water and gargling with water (spit out the water) after use. If using this inhaler for a corticosteroid preventer medication, with or without a spacer, rinse the mouth with water and spit after inhaling the last dose to reduce the risk of side-effects Option E: The client does not have to be in the supine position after administration. Proper instruction by a trained person with a placebo aerosol is essential to teach the correct inhaler technique. This should be followed subsequently by regular checks to locate any faults that may develop. Inevitably, some patients will be unable to use an MDI, and for them, spacer attachments, or dry powder inhalers are preferable since they place fewer demands on patients' skill. Even these devices, however, must be used properly to achieve a satisfactory effect.

What is a aerosol mask used for?

A mask used for the therapeutic administration of a nebulized solution, humidity, or high airflow with oxygen enrichment. It has a large-bore inlet and an exhalation port. When the required concentration needs to change during oxygen therapy, the adult aerosol mask, with the choice of 6 venturis or a multi venturi mask kit will offer the choice to suit the individual patient's requirements. The aerosol mask can be used with a nebuliser or 22mm corrugated tubing for combined oxygen therapy and humidification.

A client with COPD reports steady weight loss and being "too tired from just breathing to eat." Which of the following nursing diagnoses would be mostappropriate when planning nutritional interventions for this client? A. Altered nutrition: Less than body requirements related to fatigue B. Activity intolerance related to dyspnea C. Weight loss related to COPD D. Ineffective breathing pattern related to alveolar hypoventilation

A. Altered nutrition: Less than body requirements related to fatigue

For a male client with an endotracheal (ET) tube, which nursing action is most essential? A. Auscultating the lungs for bilateral breath sounds. B. Turning the client from side to side every 2 hours. C. Monitoring serial blood gas values every 4 hours. D. Providing frequent oral hygiene.

A. Auscultating the lungs for bilateral breath sounds For a client with an ET tube, the most important nursing action is auscultating the lungs regularly for bilateral breath sounds to ensure proper tube placement and effective oxygen delivery. Adventitious breath sounds such as wheezes and crackles are an indication of respiratory difficulties. Quick assessment allows for early detection of deterioration or improvement. Although the other options are appropriate for this client, they're secondary to ensuring adequate oxygenation. Option B: Turn the client every 2 hours. Turning mobilizes secretions and helps prevent ventilator-associated pneumonia. Auscultate the lungs for the presence of normal or adventitious breath sounds. Option C: Assess arterial blood gases (ABGs). Signs of respiratory compromise including decreasing Pao2 and increasing Paco2. Monitor oxygen saturation prior to and after suctioning using pulse oximetry. This assessment provides an evaluation of the effectiveness of therapy. Option D: Brush teeth two to three times per day with a soft toothbrush. Chlorhexidine-based rinses may also be incorporated into oral care protocols. Oral care reduces colonization of the oropharynx with respiratory pathogens that can be aspirated into the lungs.

Basilar crackles are present in a client's lungs on auscultation. The nurse knows that these are discrete, non continuous sounds that are: A. Caused by the sudden opening of alveoli. B. Usually more prominent during expiration. C. Produced by airflow across passages narrowed by secretions. D. Found primarily in the pleura.

A. Caused by the sudden opening of the alveoli Basilar crackles are usually heard during inspiration and are caused by sudden opening of the alveoli. Basilar crackles are a bubbling or crackling sound originating from the base of the lungs. They may occur when the lungs inflate or deflate. They're usually brief, and may be described as sounding wet or dry. Excess fluid in the airways causes these sounds.

When developing a discharge plan to manage the care of a client with COPD, the nurse should anticipate that the client will do which of the following? A. Develop infections easily B. Maintain current status C. Require less supplemental oxygen D. Show permanent improvement

A. Develop infections easily A client with COPD is at high risk for development of respiratory infections. In emphysema, an irritant (e.g., smoking) causes an inflammatory response. Neutrophils and macrophages are recruited and release multiple inflammatory mediators. Oxidants and excess proteases leading to the destruction of the air sacs. The protease-mediated destruction of elastin leads to a loss of elastic recoil and results in airway collapse during exhalation.

Nurse Lei, caring for a client with a pneumothorax and who has had a chest tube inserted, continues gentle bubbling in the suction control chamber. What action is appropriate? A. Do nothing, because this is an expected finding. B. Immediately clamp the chest tube and notify the physician. C. Check for an air leak because the bubbling should be intermittent. D. Increase the suction pressure so that the bubbling becomes vigorous.

A. Do nothing, because this is an expected finding. Continuous gentle bubbling should be noted in the suction control chamber. Bubbling during expiration reflects venting of pneumothorax (desired action). Bubbling usually decreases as the lung expands or may occur only during expiration or coughing as the pleural space diminishes. Option B: Chest tubes should only be clamped to check for an air leak or when changing drainage devices (according to agency policy). Clamp tubing in stepwise fashion downward toward the drainage unit if air leak continues. Isolates location of a system-centered air leak. Note: Information indicates that clamping for a suspected leak may be the only time that the chest tube should be clamped. Option C: Bubbling should be continuous and not intermittent. Seal drainage tubing connection sites securely with lengthwise tape or bands according to established policy. Prevents and corrects air leaks at connector sites. Option D: Bubbling should be gentle. Increasing the suction pressure only increases the rate of evaporation of water in the drainage system. Position drainage system tubing for an optimal function like shorten tubing or coil extra tubing on the bed, making sure tubing is not kinked or hanging below the entrance to the drainage container. Drain accumulated fluid as necessary.

A male adult client is suspected of having a pulmonary embolism. A nurse assesses the client, knowing that which of the following is a common clinical manifestation of pulmonary embolism? A. Dyspnea B. Bradypnea C. Bradycardia D. Decreased respirations

A. Dyspnea The common clinical manifestations of pulmonary embolism are tachypnea, tachycardia, dyspnea, and chest pain. PE leads to impaired gas exchange due to obstruction of the pulmonary vascular bed leading to a mismatch in the ventilation to perfusion ratio because alveolar ventilation remains the same, but pulmonary capillary blood flow decreases, effectively leading to dead space ventilation and hypoxemia.

A male client comes to the emergency department complaining of sudden onset of diarrhea, anorexia, malaise, cough, headache, and recurrent chills. Based on the client's history and physical findings, the physician suspects Legionnaires' disease. While awaiting diagnostic test results, the client is admitted to the facility and started on antibiotic therapy. What is the drug of choice for treating Legionnaires' disease? A. erythromycin (Erythrocin) B. rifampin (Rifadin) C. amantadine (Symmetrel) D. amphotericin B (Fungizone)

A. Erythromycin (Erythocin) Erythromycin is the drug of choice for treating Legionnaires' disease. Erythromycin has traditionally; it has been used for various respiratory infections (i.e., community-acquired pneumonia, Legionnaires disease), prophylaxis of neonatal conjunctivitis, and chlamydia. Erythromycin is a bacteriostatic antibiotic, which means it prevents the further growth of bacteria rather than directly destroying it. This action occurs by inhibiting protein synthesis. Option B: Rifampin may be added to the regimen if erythromycin alone is ineffective; however, it isn't administered first. Amantadine, an antiviral agent, is ineffective against Legionnaires' disease, which is caused by bacterial infection. Amphotericin B, an antifungal agent, is ineffective against Legionnaires disease because it is caused by bacteria.

Which of the following health promotion activities should the nurse include in the discharge teaching plan for a client with asthma? A. Incorporate physical exercise as tolerated into the treatment plan. B. Monitor peak flow numbers after meals and at bedtime. C. Eliminate stressors in the work and home environment. D. Use sedatives to ensure uninterrupted sleep at night.

A. Incorporate physical exercise as tolerated into the treatment plan Physical exercise is beneficial and should be incorporated as tolerated into the client's schedule. Peak flow numbers should be monitored daily, usually in the morning (before taking medication). Encourage breathing exercises and controlled breathing and relaxation. Prevents attack before it begins and increases ventilation.

Which of the following physical assessment findings would the nurse expect to find in a client with advanced COPD? A. Increased anteroposterior chest diameter B. Underdeveloped neck muscles C. Collapsed neck veins D. Increased chest excursions with respiration

A. Increased anteroposterior chest diameter Increased anteroposterior chest diameter is characteristic of advanced COPD. Air is trapped in the overextended alveoli, and the ribs are fixed in an inspiratory position. The result is the typical barrel-chested appearance. In addition, coarse crackles beginning with inspiration may be heard. Option B: Overly developed, not underdeveloped, neck muscles are associated with COPD because of their increased use in the work of breathing. Use of accessory respiratory muscles and paradoxical indrawing of lower intercostal spaces is evident (known as the Hoover sign). Option C: Distended, not collapsed, neck veins are associated with COPD as a symptom of the heart failure that the client may experience secondary to the increased workload on the heart to pump into pulmonary vasculature. In advanced disease, cyanosis, elevated jugular venous pulse (JVP), and peripheral edema can be observed. Option D: Diminished, not increased, chest excursion is associated with COPD. The sensitivity of a physical examination in detecting mild to moderate COPD is relatively poor; however, physical signs are quite specific and sensitive for severe disease. Patients with severe disease experience tachypnea and respiratory distress with simple activities.

A client is prescribed metaproterenol (Alupent) via a metered-dose inhaler (MDI), two puffs every 4 hours. The nurse instructs the client to report side effects. Which of the following are potential side effects of metaproterenol? A. Irregular heartbeat B. Constipation C. Pedal edema D. Decreased heart rate.

A. Irregular heartbeat Irregular heart rates should be reported promptly to the care provider. Metaproterenol may cause irregular heartbeat, tachycardia, or anginal pain because of its adrenergic effect on the beta-adrenergic receptors in the heart. It is not recommended for use in clients with known cardiac disorders. Metaproterenol does not cause constipation, pedal edema, or bradycardia.

A female patient suffers adult respiratory distress syndrome as a consequence of shock. The patient's condition deteriorates rapidly, and endotracheal intubation and mechanical ventilation are initiated. When the high-pressure alarm on the mechanical ventilator, alarm sounds, the nurse starts to check for the cause. Which condition triggers the high-pressure alarm? A. Kinking of the ventilator tubing. B. A disconnected ventilator tube. C. An endotracheal cuff leak. D. A change in the oxygen concentration without resetting the oxygen level alarm.

A. Kinking of the ventilator tubing Conditions that trigger the high-pressure alarm include kinking of the ventilator tubing, bronchospasm or pulmonary embolism, mucus plugging, water in the tube, coughing or biting on endotracheal tube, and the patient's being out of breathing rhythm with the ventilator. If an alarm occurs, the caregiver should always evaluate the patient before checking the ventilator. Option B: A disconnected ventilator tube would trigger the low-pressure alarm. If the pressure inside the breathing circuit drops below the Low Airway Pressure Alarm limit set on the ventilator, an audible and/or visual alarm activates. Option C: Some causes for low-pressure alarms are: the patient becomes disconnected from the ventilator circuit; inadequate inflation of the tracheostomy tube cuff; poorly fitting noninvasive masks or nasal pillows/prongs; loose circuit and tubing connections; or the patient demands higher levels of air than the ventilator is putting out. Option D: Changing the oxygen concentration without resetting the oxygen level alarm would trigger the oxygen alarm. Oxygen concentration is the amount of oxygen delivered to the patient. When the patient is not receiving added oxygen, the oxygen level will be the same as room air (21%).

A male adult patient hospitalized for treatment of a pulmonary embolism develops respiratory alkalosis. Which clinical findings commonly accompany respiratory alkalosis? A. Nausea or vomiting B. Abdominal pain or diarrhea C. Hallucinations or tinnitus D. Lightheadedness or paresthesia

A. Lightheadedness or paresthesia Correct Answer: D. Lightheadedness or paresthesia The patient with respiratory alkalosis may complain of lightheadedness or paresthesia (numbness and tingling in the arms and legs). The exact history and physical exam findings are highly variable as there are many pathologies that induce the pH disturbance. These may include acute onset dyspnea, fever, chills, peripheral edema, orthopnea, weakness, confusion, light-headedness, dizziness, anxiety, chest pain, wheezing, hemoptysis, trauma, history of central line catheter, recent surgery, history of thromboembolic disease, history of asthma, history of COPD, acute focal neurological signs, numbness, paresthesia, abdominal pain, nausea, vomiting, tinnitus, or weight loss. Option A: Nausea, vomiting, abdominal pain, and diarrhea may accompany respiratory acidosis. Following a performance predominantly relying on anaerobic glycolysis, systemic acidosis may cause vomiting as a physiological response to drain H + and thereby allow the stomach to add bicarbonate to the body Option B: Hyperchloremic acidosis is caused by the loss of too much sodium bicarbonate from the body, which can happen with severe diarrhea. In pathologies with profuse watery diarrhea, bicarbonate within the intestines is lost through the stool due to increased motility of the gut. This leads to further secretion of bicarbonate from the pancreas and intestinal mucosa, leading to net acidification of the blood from bicarbonate loss. Option C: Hallucinations and tinnitus are associated with respiratory alkalosis or any other acid-base imbalance. Respiratory alkalosis in itself is not life-threatening; however, the underlying etiology may be. Always look for and treat the source of the illness. Interventions to reduce pH directly are typically not necessary as there is no mortality benefit to this therapy.

A male client who weighs 175 lb (79.4 kg) is receiving aminophylline (Aminophylline) (400 mg in 500 ml) at 50 ml/hour. The theophylline level is reported as 6 mcg/ml. The nurse calls the physician who instructs the nurse to change the dosage to 0.45 mg/kg/hour. The nurse should: A. Question the order because it's too low. B. Question the order because it's too high. C. Set the pump at 45 ml/hour. D. Stop the infusion and have the laboratory repeat the theophylline measurement.

A. Question the order because it's too low A therapeutic theophylline level is 10 to 20 mcg/ml. The client is currently receiving 0.5 mg/kg/hour of aminophylline. Because the client's theophylline level is sub-therapeutic, reducing the dose (which is what the physician's order would do) would be inappropriate. Therefore, the nurse should question the order.

Auscultation of a client's lungs reveals crackles in the left posterior base. The nursing intervention is to: A. Repeat auscultation after asking the client to deep breathe and cough. B. Instruct the client to limit fluid intake to less than 2000 ml/day. C. Inspect the client's ankles and sacrum for the presence of edema. D. Place the client on bedrest in a semi-Fowler's position.

A. Repeat auscultation after asking the client to deep breathe and cough. Although crackles often indicate fluid in the alveoli, they may also be related to hypoventilation and will clear after a deep breath or a cough. Assess cough effectiveness and productivity. Coughing is the most effective way to remove secretions. Pneumonia may cause thick and tenacious secretions to patients.

A slightly obese female client with a history of allergy-induced asthma, hypertension, and mitral valve prolapse is admitted to an acute care facility for elective surgery. The nurse obtains a complete history and performs a thorough physical examination, paying special attention to the cardiovascular and respiratory systems. When percussing the client's chest wall, the nurse expects to elicit: A. Resonant sounds. B. Hyperresonant sounds. C. Dull sounds. D. Flat sounds.

A. Resonant Sounds When percussing the chest wall, the nurse expects to elicit resonant sounds — low-pitched, hollow sounds heard over normal lung tissue. Percussion over normal, healthy lung tissue should produce a resonant note. With the patient in an upright seated position, with the scapula protracted; percuss on the posterior chest wall; either side of the midclavicular line in the interspaces at 5cm intervals. Option B: Hyperresonant sounds indicate increased air in the lungs or pleural space; they're louder and lower pitched than resonant sounds. Although hyperresonant sounds occur in such disorders as emphysema and pneumothorax, they may be normal in children and very thin adults. Option C: Dull sounds, normally heard only over the liver and heart, may occur over dense lung tissue, such as from consolidation or a tumor. Dull sounds are thudlike and of medium pitch. Dull percussive sounds are indicative of abnormal lung density. Likely indicating: atelectasis, tumour, pleural effusion, lobar pneumonia Option D: Flat sounds, soft and high-pitched, are heard over airless tissue and can be replicated by percussing the thigh or a bony structure. Percussion produces sounds on a spectrum from flat to dull depending on the density of the underlying tissue.

If a client continues to hypoventilate, the nurse will continually assess for a complication of: A. Respiratory acidosis B. Respiratory alkalosis C. Metabolic acidosis D. Metabolic alkalosis

A. Respiratory acidosis Respiratory acidosis represents an increase in the acid component, carbon dioxide, and an increase in the hydrogen ion concentration (decreased pH) of the arterial blood. The respiratory centers in the pons and medulla control alveolar ventilation. Chemoreceptors for PCO2, PO2, and pH regulate ventilation. Central chemoreceptors in the medulla are sensitive to changes in the pH level. A decreased pH level influences the mechanics of ventilation and maintains proper levels of carbon dioxide and oxygen. When ventilation is disrupted, arterial PCO2 increases and an acid-base disorder develops. Option B: In almost every scenario, respiratory alkalosis is induced by a process involving hyperventilation. These include central causes, hypoxemic causes, pulmonary causes, and iatrogenic causes. Central sources are a head injury, stroke, hyperthyroidism, anxiety-hyperventilation, pain, fear, stress, drugs, medications such as salicylates, and various toxins. Hypoxic stimulation leads to hyperventilation in an attempt to correct hypoxia at the expense of a CO2 loss. Option C: Hydrogen ion concentration is determined by acid ingestion, acid production, acid excretion, and renal and GI bicarbonate losses. Buffers such as bicarbonate minimize significant pH alterations. Further classification of metabolic acidosis is based on the presence or absence of an anion gap, or concentration of unmeasured serum anions. Option D: In general, the causes can be narrowed down to an intracellular shift of hydrogen ions, gastrointestinal (GI) loss of hydrogen ions, excessive renal hydrogen ion loss, retention or addition of bicarbonate ions, or volume contraction around a constant amount of extracellular bicarbonate known as contraction alkalosis. All of which leads to the net result of increased levels of bicarbonate in the blood.

Nurse Oliver is caring for a client immediately after removal of the endotracheal tube. The nurse reports which of the following signs immediately if experienced by the client? A. Stridor B. Occasional pink-tinged sputum C. A few basilar lung crackles on the right D. Respiratory rate 24 breaths/min

A. Stridor The nurse reports stridor to the physician immediately. This is a high-pitched, coarse sound that is heard with the stethoscope over the trachea. Stridor indicates airway edema and places the client at risk for airway obstruction. Post-extubation stridor is uncommon and seen only in less than 10% of unselected critically ill patients and correlates with increased rates of reintubation, prolonged duration of mechanical ventilation, and longer length of ICU stay. Options B, C, and D are not signs that require immediate notification of the physician. Option B: A pink-tinged sputum is normal after removal of the endotracheal tube. Ensure adequate secretion management, encourage coughing and deep breathing, maintain airway hydration, and patent central airway. The immediate post-extubation phase should be managed as carefully as the ventilated phase because the first 24 hours post-extubation are difficult and tenuous. Option C: After the removal of the ETT, suction the oral cavity and ask the patient to take a deep breath and cough out all secretions. Frequent airway suction should be considered to prevent re-intubation. There may be a few crackles upon auscultation. Option D: 24 breaths/min is a normal respiratory rate. The patient should be placed on supplemental oxygen afterward. Ensure adequate oxygenation; consider nasal cannula, oxygen mask, full face mask, venturi mask as appropriate to ensure good O2 supply. In recent times, high flow oxygen systems have gained popularity in selected patients with hypoxemic respiratory failure and studies have shown to reduce the re-intubation rate.

The nurse is planning to teach a client with COPD how to cough effectively. Which of the following instructions should be included? A. take a deep abdominal breath, bend forward, and cough 3-4 times on exhalation B. lie flat on back, splint the thorax, take 2 deep breaths and cough C. take several rapid, shallow breaths and then cough forcefully D. assume a side-lying position, extend the arm over the head, and alternate deep breathing with coughing.

A. Take a deep abdominal breath, bend forward, and cough 3-4 times on exhalation The goal of effective coughing is to conserve energy, facilitate the removal of secretions, and minimize airway collapse. The client should assume a sitting position with feet on the floor if possible. The client should bend forward slightly and, using pursed-lip breathing, exhale. After resuming an upright position, the client should use abdominal breathing to slowly and deeply inhale. After repeating this process 3 or 4 times, the client should take a deep abdominal breath, bend forward and cough 3 or 4 times upon exhalation ("huff" cough).

Which of the following ABG abnormalities should the nurse anticipate in a client with advanced COPD? A. increased PaCO2 B. increased PaO2 C. increased pH D. increased oxygen saturation

A. increased PaCO2 As COPD progresses, the client typically develops increased PaCO2 levels and decreased PaO2 levels. This results in decreased pH and decreased oxygen saturation. These changes are the result of air trapping and hypoventilation. Arterial blood gas (ABG) analysis provides the best clues as to acuteness and severity of disease exacerbation.

Which of the following is a priority goal for the client with COPD? A. Maintaining functional ability B. Minimizing chest pain C. Increasing carbon dioxide levels in the blood D. Treating infectious agents

A. maintaining functional ability

A nurse is caring for a male client with emphysema who is receiving oxygen. The nurse assesses the oxygen flow rate to ensure that it does not exceed: A. 1 L/min B. 2 L/min C. 6 L/min D. 10 L/min

B. 2 L/min Oxygen is used cautiously and should not exceed 2 L/min. Because of the long-standing hypercapnia that occurs in emphysema, the respiratory drive is triggered by low oxygen levels rather than increased carbon dioxide levels, as is the case in a normal respiratory system. Supplemental oxygen can successfully reach the alveoli in these lungs, which prevents this vasoconstriction and thereby increases perfusion and improves gas exchange, thus resulting in improvement of hypoxemia. Option A: Routine supplemental oxygen does not improve the quality of life or clinical outcomes in stable patients. Continuous long-term, i.e., longer than 15 hours of supplemental oxygen is recommended in patients with COPD with PaO2 less than 55 mmHg (or oxygen saturation less than 88%) or PaO2 less than 59 mm Hg in case of cor pulmonale. Option C: Oxygen therapy has shown to increase the survival of these patients with severe resting hypoxemia. For those who desaturate with exercise, intermittent oxygen will help. The goal is to maintain oxygen saturation greater than 90%. Option D: Excessive correction of hypoxia in a patient with longstanding COPD can sometimes lead to hypercapnia. This is due to the loss of compensatory vasoconstriction with an ineffective gas exchange as there is a loss of hypoxic drive for ventilation. Also, increased oxyhemoglobin decreases the uptake of carbon dioxide due to the Haldane effect.

A female adult client has a tracheostomy but doesn't require continuous mechanical ventilation. When weaning the client from the tracheostomy tube, the nurse initially should plug the opening in the tube for: A. 15 to 60 seconds. B. 5 to 20 minutes. C. 30 to 40 minutes. D. 45 to 60 minutes.

B. 5-20 minutes Initially, the nurse should plug the opening in the tracheostomy tube for 5 to 20 minutes, then gradually lengthen this interval according to the client's respiratory status. Tracheal plugging is ordered by the doctor to help the client wean off an artificial airway. Plugging covers the opening of the trach tube in the throat, and allowing her to breathe through her nose mouth. Plugging will also help make the sound of her voice stronger.

A client is admitted to the hospital with acute bronchitis. While taking the client's VS, the nurse notices he has an irregular pulse. The nurse understands that cardiac arrhythmias in chronic respiratory distress are usually the result of: A. Respiratory acidosis B. A build-up of carbon dioxide C. A build-up of oxygen without adequate expelling of carbon dioxide. D. An acute respiratory infection.

B. A build up of carbon dioxide The arrhythmias are caused by a build-up of carbon dioxide and not enough oxygen so that the heart is in a constant state of hypoxia. The majority of arrhythmias observed in these patients appeared to take the form of premature ventricular and/or supraventricular beats and less frequently of atrial fibrillation and/or attacks of supraventricular paroxysmal tachycardia. Cardiac rhythm alterations were observed using Holter monitoring in 70-90% of patients. No cardiac rhythm disorder is specific to this pathological condition.

Nurse Reese is caring for a client hospitalized with acute exacerbation of chronic obstructive pulmonary disease. Which of the following would the nurse expect to note on assessment of this client? A. Hypocapnia B. A hyperinflated chest noted on the chest x-ray. C. Increased oxygen saturation with exercise. D. A widened diaphragm noted on the chest x-ray.

B. A hyperinflated chest noted on the CXR Clinical manifestations of chronic obstructive pulmonary disease (COPD) include hypoxemia, hypercapnia, dyspnea on exertion and at rest, oxygen desaturation with exercise, and the use of accessory muscles of respiration. Chest x-rays reveal a hyperinflated chest and a flattened diaphragm if the disease is advanced. Hyperinflation of the lungs is often seen on imaging studies and occurs due to air trapping from airway collapse during exhalation. Option A: The inability to fully exhale also causes elevations in carbon dioxide (CO2) levels. As the disease progresses, impairment of gas exchange is often seen. The reduction in ventilation or increase in physiologic dead space leads to CO2 retention. Patients may have acute respiratory failure and physical findings of hypoxemia and hypercapnia. Option C: Acute exacerbations of COPD are common and usually occur due to a trigger (e.g., bacterial or viral pneumonia, environmental irritants). There is an increase in inflammation and air trapping often requiring corticosteroid and bronchodilator treatment. Acute exacerbations of COPD usually present with increased dyspnea, productive cough, and wheezing. Option D: Radiographic imaging includes a chest x-ray and computed tomography (CT). Chest x-rays may show hyperinflation, flattening of the diaphragm, and increased anterior-posterior diameter. In cases of chronic bronchitis, bronchial wall thickening may be present.

On auscultation, which finding suggests a right pneumothorax? A. Bilateral inspiratory and expiratory crackles. B. Absence of breaths sound in the right thorax. C. Inspiratory wheezes in the right thorax. D. Bilateral pleural friction rub.

B. Absence of breath sounds in the right thorax In pneumothorax, the alveoli are deflated and no air exchange occurs in the lungs. Therefore, breath sounds in the affected lung field are absent. A pneumothorax is defined as a collection of air outside the lung but within the pleural cavity. It occurs when air accumulates between the parietal and visceral pleura inside the chest. The air accumulation can apply pressure on the lung and make it collapse. The degree of collapse determines the clinical presentation of pneumothorax. None of the other options are associated with pneumothorax. Option A: Bilateral crackles may result from pulmonary congestion. Pneumonia is an infection in the lungs. It may be in one or both lungs. The infection causes air sacs in the lungs to become pus-filled and inflamed. This causes a cough, difficulty breathing, and crackles. Pneumonia may be mild or life-threatening. Option C: Inspiratory wheezes may signal asthma. Asthma is a heterogeneous syndrome characterized by variable, reversible airway obstruction and abnormally increased responsiveness (hyperreactivity) of the airways to various stimuli. The syndrome is characterized by wheezing, chest tightness, dyspnea, and/or cough, and results from widespread contraction of tracheobronchial smooth muscle (bronchoconstriction), hypersecretion of mucus, and mucosal edema, all of which narrow the caliber of the airways. Option D: A pleural friction rub may indicate pleural inflammation. Auscultation of a pleural friction rub can occur when the normally smooth surfaces of the visceral and parietal pleura become roughened by inflammation. A pleural friction rub is an adventitious breath sound heard on auscultation of the lung. The pleural rub sound results from the movement of inflamed and roughened pleural surfaces against one another during movement of the chest wall. This sound is non-musical, and described as "grating," "creaky," or "the sound made by walking on fresh snow."

A male patient has a sucking stab wound to the chest. Which action should the nurse take first? A. Drawing blood for a hematocrit and hemoglobin level. B. Applying a dressing over the wound and taping it on three sides. C. Preparing a chest tube insertion tray. D. Preparing to start an I.V. line.

B. Applying a dressing over the wound and taping it on 3 sides The nurse immediately should apply a dressing over the stab wound and tape it on three sides to allow air to escape and to prevent tension pneumothorax (which is more life-threatening than an open chest wound). The nurse may use a first aid device called a chest seal or improvise with the packaging sterile dressings coming in. Peel open the packaging and tape the entire plastic portion over the wound. Only after covering and taping the wound should the nurse draw blood for laboratory tests, assist with chest tube insertion, and start an I.V. line.

Rhea, confused and short breath, is brought to the emergency department by a family member. The medical history reveals chronic bronchitis and hypertension. To learn more about the current respiratory problem, the doctor orders a chest x-ray and arterial blood gas (ABG) analysis. When reviewing the ABG report, the nurses see many abbreviations. What does a lowercase "a" in ABG value present? A. Acid-base balance B. Arterial Blood C. Arterial oxygen saturation D. Alveoli

B. Arterial Blood A lowercase "a" in an ABG value represents arterial blood. For instance, the abbreviation PaO2 refers to the partial pressure of oxygen in arterial blood. Arterial blood gas analysis can be used to assess gas exchange and acid base status as well as to provide immediate information about electrolytes.

Nurse Maureen has assisted a physician with the insertion of a chest tube. The nurse monitors the client and notes fluctuation of the fluid level in the water seal chamber after the tube is inserted. Based on this assessment, which action would be appropriate? A. Inform the physician. B. Continue to monitor the client. C. Reinforce the occlusive dressing. D. Encourage the client to deep breathe.

B. Continue to monitor the client The presence of fluctuation of the fluid level in the water seal chamber indicates a patent drainage system. With normal breathing, the water level rises with inspiration and falls with expiration. Fluctuation stops if the tube is obstructed, if a dependent loop exists, if the suction is not working properly, or if the lung has re-expanded.

When caring for a male patient who has just had a total laryngectomy, the nurse should plan to: A. Encourage oral feeding as soon as possible. B. Develop an alternative communication method. C. Keep the tracheostomy cuff fully inflated. D. Keep the patient flat in bed.

B. Develop an alternative communication method

An emergency room nurse is assessing a male client who has sustained a blunt injury to the chest wall. Which of these signs would indicate the presence of a pneumothorax in this client? A. A low respiratory rate. B. Diminished breath sounds. C. The presence of a barrel chest. D. A sucking sound at the site of injury.

B. Diminished breath sounds This client has sustained a blunt or a closed chest injury. Basic symptoms of a closed pneumothorax are shortness of breath and chest pain. A larger pneumothorax may cause tachypnea, cyanosis, diminished breath sounds, and subcutaneous emphysema. Hyperresonance also may occur on the affected side. A pneumothorax is defined as a collection of air outside the lung but within the pleural cavity. It occurs when air accumulates between the parietal and visceral pleura inside the chest. The air accumulation can apply pressure on the lung and make it collapse.

Assessing a client who has developed atelectasis postoperatively, the nurse will most likely find: A. A flushed face. B. Dyspnea and pain. C. Decreased temperature. D. Severe cough and no pain.

B. Dyspnea and pain Atelectasis is a collapse of the alveoli due to obstruction or hypoventilation. Clients become short of breath, have a high temperature, and usually experience severe pain but do not have a severe cough. The shortness of breath is a result of decreased oxygen-carbon dioxide exchange at the alveolar level. Postoperative atelectasis typically occurs within 72 hours of general anesthesia and is a well-known postoperative complication.

A client states that the physician said the tidal volume is slightly diminished and asks the nurse what this means. The nurse explains that the tidal volume is the amount of air: A. Exhaled forcibly after a normal expiration. B. Exhaled after there is a normal inspiration. C. Trapped in the alveoli that cannot be exhaled. D. Forcibly inspired over and above a normal respiration.

B. Exhaled forcibly after a normal expiration Tidal volume (TV) is defined as the amount of air exhaled after a normal inspiration. Tidal volume is the amount of air that moves in or out of the lungs with each respiratory cycle. It measures around 500 mL in an average healthy adult male and approximately 400 mL in a healthy female. It is a vital clinical parameter that allows for proper ventilation to take place. Option A: The expiratory reserve volume (ERV), about 1,200 mL, is the additional air that can be forcibly exhaled after the expiration of a normal tidal volume. When a person breathes in, oxygen from the surrounding atmosphere enters the lungs. It then diffuses across the alveolar-capillary interface to reach arterial blood. At the same time, carbon dioxide continuously forms as long as metabolism takes place. Expiration occurs to expel carbon dioxide and prevent it from accumulating in the body. Option C: Residual volume (RV), about 1,200 mL, is the volume of air still remaining in the lungs after the expiratory reserve volume is exhaled. When emphysema develops, the alveoli and lung tissue are destroyed. With this damage, the alveoli cannot support the bronchial tubes. The tubes collapse and cause an "obstruction" (a blockage), which traps air inside the lungs. Option D: The inspiratory reserve volume (IRV), about 3,100 mL, is the additional air that can be forcibly inhaled after the inspiration of a normal tidal volume. The volume of air occupying the lungs at different phases of the respiratory cycle subdivides into four volumes and four capacities. The four lung volumes are inspiratory reserve volume (IRV), expiratory reserve volume (ERV), tidal volume (V), and residual volume (RV), while the four lung capacities include total lung capacity (TLC), vital capacity (VC), inspiratory capacity (IC), and functional residual capacity (FRC).

While changing the tapes on a tracheostomy tube, the male client coughs and the tube is dislodged. The initial nursing action is to: A. Call the physician to reinsert the tube. B. Grasp the retention sutures to spread the opening. C. Call the respiratory therapy department to reinsert the tracheotomy. D. Cover the tracheostomy site with a sterile dressing to prevent infection.

B. Grasp the retention sutures to spread the opening If the tube is dislodged accidentally, the initial nursing action is to grasp the retention sutures and spread the opening. The stay suture (if present) or tracheal dilator may be used to help keep the stoma open if necessary. If agency policy permits, the nurse then attempts immediately to replace the tube. Options A and C will delay treatment in this emergency situation. Option A: When a tube is dislodged, a nurse is generally the first responder. At the first sign of a possible dislodged tube, another caregiver should send another individual to urgently summon a physician. Option C: If the tracheostomy is new, only a physician should reinsert the tube, and a nurse or respiratory therapist should never attempt to reposition the tube. This is because it takes time for the tract to form, and repositioning before the tract has formed can lead to complications as severe as those caused by the failure to act. Option D: Covering the tracheostomy site will block the airway. A dislodged tube also calls for immediate attempts at manual ventilation, and suction with a solution of sodium chloride. This will rule out a mucus plug. Once this is done, to prevent brain damage the nurse should immediately deflate the tracheostomy cuff and take out the tracheostomy tube.

A nurse is preparing to obtain a sputum specimen from a male client. Which of the following nursing actions will facilitate obtaining the specimen? A. Limiting fluid. B. Having the client take deep breaths. C. Asking the client to spit into the collection container. D. Asking the client to obtain the specimen after eating.

B. Having the client take deep breaths To obtain a sputum specimen, the client should rinse the mouth to reduce contamination, breathe deeply, and then cough into a sputum specimen container. To cough deeply from the lungs, the client might need to take three deep breaths before he coughs forcefully. Option A: Sputum can be thinned by fluids or by a respiratory treatment such as inhalation of nebulized saline or water. Drinking plenty of fluids can help loosen the secretions and make it easier to cough up sputum. The doctor may ask the client to rinse out his mouth with clear water to help get rid of any other bacteria and extra saliva. Option C: The client should be encouraged to cough and not spit so as to obtain sputum. To be sure the test is accurate, the client must cough up sputum from deep inside the lungs. Sputum from the lungs is usually thick and sticky. Saliva comes from your mouth and is watery and thin. Do not collect saliva. Option D: The optimal time to obtain a specimen is on rising in the morning. As soon as the client wakes up in the morning (before he eats or drinks anything), he should brush his teeth and rinse his mouth with water. Do not use mouthwash.

A client with COPD has developed secondary polycythemia. Which nursing diagnosis would be included in the plan of care because of the polycythemia? A. Fluid volume deficit related to blood loss. B. Impaired tissue perfusion related to thrombosis. C. Activity intolerance related to dyspnea. D. Risk for infection related to suppressed immune response.

B. Impaired tissue perfusion related to thrombosis Chronic hypoxia associated with COPD may stimulate excessive RBC production (polycythemia). This results in increased blood viscosity and the risk of thrombosis. The other nursing diagnoses are not applicable in this situation. The most common causes of secondary polycythemia include obstructive sleep apnea, obesity hypoventilation syndrome, and chronic obstructive pulmonary disease (COPD).

The nurse would anticipate which of the following ABG results in a client experiencing a prolonged, severe asthma attack? A. Decreased PaCO2, increased PaO2, and decreased pH. B. Increased PaCO2, decreased PaO2, and decreased pH. C. Increased PaCO2, increased PaO2, and increased pH. D. Decreased PaCO2, decreased PaO2, and increased pH.

B. Increased PaCO2, decreased PaO2, and decreased pH. As the severe asthma attack worsens, the client becomes fatigued and alveolar hypotension develops. This leads to carbon dioxide retention and hypoxemia. The client develops respiratory acidosis. Therefore, the PaCO2 level increases, the PaO2 level decreases, and the pH decreases, indicating acidosis.

A nurse is caring for a male client with acute respiratory distress syndrome. Which of the following would the nurse expect to note in the client? A. Pallor B. Low arterial PaO2 C. Elevated arterial PaO2 D. Decreased respiratory rate

B. Low arterial PaO2 The earliest clinical sign of acute respiratory distress syndrome is an increased respiratory rate. Breathing becomes labored, and the client may exhibit air hunger, retractions, and cyanosis. Arterial blood gas analysis reveals increasing hypoxemia, with a PaO2 lower than 60 mm Hg. Despite 100% oxygen, patients have low oxygen saturation.

A black client with asthma seeks emergency care for acute respiratory distress. Because of this client's dark skin, the nurse should assess for cyanosis by inspecting the: A. Lips B. Mucous membranes C. Nail beds D. Earlobes

B. Mucous membranes (lips, gums, and around the eyes)

Dr. Jones prescribes albuterol sulfate (Proventil) for a patient with newly diagnosed asthma. When teaching the patient about this drug, the nurse should explain that it may cause: A. Nasal congestion B. Nervousness C. Lethargy D. Hyperkalemia

B. Nervousness Albuterol may cause nervousness. The primary adverse effects of albuterol therapy are tremors and nervousness, mostly seen in children who are 2 to 6 years of age, though can be seen at any age. Tremors are the result of activation of the beta-2 receptors found on the motor nerve terminals which increases intracellular cAMP. These side effects occur in approximately one in every five patients. Other adverse effects of albuterol include tremor, dizziness, headache, tachycardia, palpitations, hypertension, heartburn, nausea, vomiting and muscle cramps. Option A: The inhaled form of the drug may cause dryness and irritation of the nose and throat, not nasal congestion. Monitoring parameters for albuterol include forced expiratory volume, peak flow, blood pressure, heart rate, central nervous system stimulation, serum potassium, serum glucose, and asthma symptoms. Option C: Other side effects include insomnia and nausea, which occur in approximately 1 in every ten patients. Less common adverse effects may include fever, bronchospasm, vomiting, headache, dizziness, cough, allergic reactions, otitis media, epistaxis, increased appetite, urinary tract infections, dry mouth, gas, hyperhidrosis, pain, dyspepsia, hyperactivity, chills, lymphadenopathy, ocular pruritus, sweating, conjunctivitis, and dysphonia. Option D: Albuterol also has been shown to increase blood pressure and may cause hypokalemia. Increased blood glucose concentrations and prolonged QTc interval and ST-segment depression have occurred, although rarely.

A male client abruptly sits up in bed, reports having difficulty breathing and has an arterial oxygen saturation of 88%. Which mode of oxygen delivery would most likely reverse the manifestations? A. Simple mask B. Non-rebreather mask C. Face tent D. Nasal cannula

B. Non-rebreather mask A non-rebreather mask can deliver levels of the fraction of inspired oxygen (FIO2) as high as 100%. Other modes — simple mask, face tent, and nasal cannula — deliver lower levels of FIO2. Non-rebreathing masks have a bag attached to the mask known as a reservoir bag, which inhalation draws from to fill the mask through a one-way valve and features ports at each side for exhalation, resulting in an ability to provide the patient with 100% oxygen at a higher LPM flow rate. Option A: Face masks can be generally divided into simple facemasks, air-entrainment masks, and non-rebreathers. A simple facemask is a mask with no bag attached, which delivers oxygen at 5 to 8 LPM. A disadvantage of this and other full face masks is the inability of the patient to eat, drink, or easily communicate while using such a device. Option C: Face tents are used to provide a controlled concentration of oxygen and increase moisture for patients who have facial burn or a broken nose, or who are claustrophobic. The mask covers the nose and mouth and does not create a seal around the nose. It can provide 28% to 100% O2 Flow meter should be set to deliver O2 at a minimum of 15 L/min. It is difficult to achieve high levels of oxygenation with this mask. Option D: Nasal cannula is a thin tube, often affixed behind the ears and used to deliver oxygen directly to the nostrils from a source connected with tubing. This is the most common method of delivery for home use and provides flow rates of 2 to 6 liters per minute (LPM) comfortably, allowing the delivery of oxygen while maintaining the patient's ability to utilize his or her mouth to talk, eat, etc.

The physician has scheduled a client for a left pneumonectomy. The position that will most likely be ordered postoperatively for his is the: A. Nonoperative side or back B. Operative side or back C. Back only D. Back or either side.

B. Operative side or back Following pneumonectomy, the client is positioned on the operative side to allow the fluid left in the lung space to consolidate and avoid the heart from shifting to the operative side. Pneumonectomy is defined as the surgical removal of the entire lung. Extrapleural pneumonectomy is an expanded procedure that also involves resection of parietal and visceral pleura, ipsilateral hemidiaphragm, pericardium, and mediastinal lymph nodes.

A female client comes into the emergency room complaining of SOB and pain in the lung area. She states that she started taking birth control pills 3 weeks ago and that she smokes. Her VS are: 140/80, P 110, R 40. The physician orders ABG's, results are as follows: pH: 7.50; PaCO2 29 mm Hg; PaO2 60 mm Hg; HCO3- 24 mEq/L; SaO2 86%. Considering these results, the first intervention is to: A. Begin mechanical ventilation. B. Place the client on oxygen. C. Give the client sodium bicarbonate. D. Monitor for pulmonary embolism.

B. Place the client on oxygen The pH (7.50) reflects alkalosis, and the low PaCO2 indicates the lungs are involved. The client should immediately be placed on oxygen via mask so that the SaO2 is brought up to 95%. Encourage slow, regular breathing to decrease the amount of CO2 she is losing.

Nurse Reynolds caring for a client with a chest tube turns the client to the side, and the chest tube accidentally disconnects. The initial nursing action is to: A. Call the physician. B. Place the tube in a bottle of sterile water. C. Immediately replace the chest tube system. D. Place a sterile dressing over the disconnection site.

B. Place the tube in a bottle of sterile water If the chest drainage system is disconnected, the end of the tube is placed in a bottle of sterile water held below the level of the chest. Anchor thoracic catheter to the chest wall and provide an extra length of tubing before turning or moving the patient. Prevents thoracic catheter dislodgement or tubing disconnection and reduces pain and discomfort associated with pulling or jarring of tubing. Option A: The physician may need to be notified, but this is not the initial action. Observe for signs of respiratory distress. If possible, reconnect thoracic catheter to tubing or suction, using clean technique. Option C: The system is replaced if it breaks or cracks or if the collection chamber is full. Pneumothorax may recur, requiring prompt intervention to prevent fatal pulmonary and circulatory impairment. Option D: Placing a sterile dressing over the disconnection site will not prevent complications resulting from the disconnection. If the catheter is dislodged from the chest, cover the insertion site immediately with petrolatum dressing and apply firm pressure. Notify the physician at once.

A male client is asking the nurse a question regarding the Mantoux test for tuberculosis. The nurse should base her response on the fact that the: A. Area of redness is measured in 3 days and determines whether tuberculosis is present. B. Skin test doesn't differentiate between active and dormant tuberculosis infection. C. Presence of a wheal at the injection site in 2 days indicates active tuberculosis. D. Test stimulates a reddened response in some clients and requires a second test in 3 months.

B. Skin test doesn't differentiate between active and dormant TB The Mantoux test doesn't differentiate between active and dormant infections. If a positive reaction occurs, a sputum smear and culture as well as a chest X-ray are necessary to provide more information. According to CDC, this test is performed using the 'Mantoux technique,' which is injecting 0.1 mL of a solution containing 5 units of tuberculin purified protein derivative into the inner surface of the forearm through the intradermal route. It should be administered two or more than 2 inches from the elbow, wrist, or any other injection site.

A nurse performs an admission assessment on a female client with a diagnosis of tuberculosis. The nurse reviews the result of which diagnosis test that will confirm this diagnosis? A. Bronchoscopy B. Sputum culture C. Chest x-ray D. Tuberculin skin test

B. Sputum Culture Tuberculosis is definitively diagnosed through culture and isolation of Mycobacterium tuberculosis. Mycobacterial culture is the gold standard for diagnosis. Mycobacterial culture should be performed on both the solid and liquid medium. Liquid media culture can detect very low bacterial load and is considered a gold standard. Culture essential for drug susceptibility testing. A presumptive diagnosis is made based on a tuberculin skin test, a sputum smear that is positive for acid-fast bacteria, a chest x-ray, and histological evidence of granulomatous disease on biopsy. Active tuberculosis is diagnosed by isolating Mycobacterium tuberculosis complex bacilli from bodily secretions. Option A: If all measures fail to obtain a sputum sample, a fiberoptic bronchoscopy with bronchoalveolar lavage can be performed with or without a transbronchial biopsy. Bronchoscopy can also be performed in high clinical suspicion with negative sputum studies and to rule out an alternative diagnosis. Option C: Primary tuberculosis often causes middle and lower lung field opacities associated with mediastinal adenopathy. Whereas secondary tuberculosis commonly involves upper lobes, causing opacities, cavities, or fibrotic scar tissue. Option D: The Mantoux test is a two-part test consisting of an intradermal injection of .1ml purified protein derivative and observing for induration 48-72 hours. The patient's risk of exposure is taken into consideration when interpreting the result. Patients are then classified into three groups based on the size of the induration and the risk of exposure.

The nurse assesses a male client's respiratory status. Which observation indicates that the client is experiencing difficulty breathing? A. Diaphragmatic breathing B. Use of accessory muscles C. Pursed-lip breathing D. Controlled breathing

B. Use of accessory muscles

An oxygen delivery system is prescribed for a male client with chronic obstructive pulmonary disease to deliver a precise oxygen concentration. Which of the following types of oxygen delivery systems would the nurse anticipate to be prescribed? A. Face tent B. Venturi mask C. Aerosol mask D. Tracheostomy collar

B. Venturi Mask The face tent, aerosol mask, and tracheostomy collar are also high-flow oxygen delivery systems but most often are used to administer high humidity.

The client with asthma should be taught which of the following is one of the most common precipitating factors of an acute asthma attack? A. Occupational exposure to toxins. B. Viral respiratory infections. C. Exposure to cigarette smoke. D. Exercising in cold temperatures

B. Viral respiratory infections The most common precipitator of asthma attacks is viral respiratory infection. Clients with asthma should avoid people who have the flu or a cold and should get yearly flu vaccinations. Asthma is a condition of acute, fully reversible airway inflammation, often following exposure to an environmental trigger. The pathological process begins with the inhalation of an irritant (e.g., cold air) or an allergen (e.g., pollen), which then, due to bronchial hypersensitivity, leads to airway inflammation and an increase in mucus production. This leads to a significant increase in airway resistance, which is most pronounced on expiration.

When teaching a client with COPD to conserve energy, the nurse should teach the client to lift objects: A. while inhaling through an open mouth B. while exhaling through pursed lips C. after exhaling but before inhaling D. while taking a deep breath and holding it

B. While exhaling through pursed lips Exhaling requires less energy than inhaling. Therefore, lifting while exhaling saves energy and reduces perceived dyspnea. When one practices regularly, breathing exercises can help exert oneself less during daily activities. They can also potentially aid in return to exercising, which can lead to feeling more energetic overall.

A client's arterial blood gas levels are as follows: pH 7.31; PaO2 80 mm Hg, PaCO2 65 mm Hg; HCO3- 36 mEq/L. Which of the following signs or symptoms would the nurse expect? A. cyanosis B. flushed skin C. Irritability D. Anxiety

B. flushed skin The high PaCO2 level causes flushing due to vasodilation. The client also becomes drowsy and lethargic because carbon dioxide has a depressant effect on the CNS. On the contrary, chronic respiratory acidosis may be caused by COPD where there is a decreased responsiveness of the reflexes to states of hypoxia and hypercapnia.

A client has been taking flunisolide (Aerobid), two inhalations a day, for treatment of asthma. He tells the nurse that he has painful, white patches in his mouth. Which response by the nurse would be the most appropriate? A. "This is an anticipated side-effect of your medication. It should go away in a couple of weeks." B. "You are using your inhaler too much and it has irritated your mouth." C. "You have developed a fungal infection from your medication. It will need to be treated with an antibiotic." D. "Be sure to brush your teeth and floss daily. Good oral hygiene will treat this problem."

C. "You have developed a fungal infection from your medication. It will need to be treated with an antibiotic." Use of oral inhalant corticosteroids, such as flunisolide, can lead to the development of oral thrush, a fungal infection. Oral candidiasis (thrush) is another common complaint among users of inhaled corticosteroids. This risk increases in elderly patients and patients who are also taking oral steroids, high dose ICS, or antibiotics. Option A: Once developed, thrush must be treated by antibiotic therapy; it will not resolve on its own. It is advisable to have the patient rinse their mouth out after ICS use to prevent oral candidiasis. Treatments for candidiasis include clotrimazole, miconazole, and nystatin. Option B: Fungal infections can develop even without overuse of the Corticosteroid inhaler. Attention to dosage is required as the amount of Candida increased with dose of fluticasone. Gargling with a 1:50 dilution of amphotericin B is effective in treating oral candidiasis of asthmatic patients treated with inhaled steroids. Option D: Although good oral hygiene can help prevent the development of a fungal infection, it cannot be used alone to treat the problem. Most cases of oral thrush will clear up in a couple of weeks. In general, a single dose of antifungal medication may be enough to cure the infection.

A nurse is suctioning fluids from a male client via a tracheostomy tube. When suctioning, the nurse must limit the suctioning time to a maximum of: A. 1 minute B. 5 seconds C. 10 seconds D. 30 seconds

C. 10 seconds Hypoxemia can be caused by prolonged suctioning, which stimulates the pacemaker cells in the heart. A vasovagal response may occur, causing bradycardia. The nurse must preoxygenate the client before suctioning and limit the suctioning pass to 10 seconds. It is of particular importance for patients with mechanical ventilators, endotracheal tube (ET) intubations, tracheostomies, or other airway adjuncts. Clearance of airway secretions is a normal process and is critical to the prevention of respiratory infections, atelectasis, and preservation of airway patency.

An acceleration in oxygen dissociation from hemoglobin, and thus oxygen delivery to the tissues, is caused by: A. A decreasing oxygen pressure in the blood. B. An increasing carbon dioxide pressure in the blood. C. A decreasing oxygen pressure and/or an increasing carbon dioxide pressure in the blood. D. An increasing oxygen pressure and/or a decreasing carbon dioxide pressure in the blood.

C. A decreasing oxygen pressure and/or an increasing carbon dioxide pressure in the blood. The lower the PO2 and the higher the PCO2, the more rapidly oxygen dissociated from the oxyhemoglobin molecule. Factors that contribute to a right-shift in the oxygen dissociation curve and favor the unloading of oxygen correlate with exertion. These include increased body temperature, decreased pH (due to increased production of CO2), and increased 2,3-BPG. (Figure) This right shift of the oxyhemoglobin curve can be viewed as an adaptation for physical exertion.

A cyanotic client with an unknown diagnosis is admitted to the E.R. In relation to oxygen, the first nursing action would be to: A. Wait until the client's lab work is done. B. Not administer oxygen unless ordered by the physician. C. Administer oxygen at 2 L flow per minute. D. Administer oxygen at 10 L flow per minute and check the client's nail beds.

C. Administer oxygen at 2 L flow per minute Administer oxygen at 2 L/minute and no more, for if the client is emphysemic and receives too high a level of oxygen, he will develop CO2 narcosis and the respiratory system will cease to function. With prolonged oxygen therapy there is an increase in blood oxygen level, which suppresses peripheral chemoreceptors; depresses ventilator drive and increase in PCO2. high blood oxygen level may also disrupt the ventilation: perfusion balance (V/Q) and cause an increase in dead space to tidal volume ratio and increase in PCO2.

A fifty-year-old client has a tracheostomy and requires tracheal suctioning. The first intervention in completing this procedure would be to: A. Change the tracheostomy dressing. B. Provide humidity with a trach mask. C. Apply oral or nasal suction. D. Deflate the tracheal cuff.

C. Apply oral or nasal suction Before deflating the tracheal cuff, the nurse will apply oral or nasal suction to the airway to prevent secretions from falling into the lung. Dressing change and humidity do not relate to suctioning. Airway suctioning is a procedure routinely done in most care settings, including acute care, sub-acute care, long-term care, and home settings. Suctioning is performed when the patient is unable to effectively move secretions from the respiratory tract.

A male client with chronic obstructive pulmonary disease (COPD) is recovering from a myocardial infarction. Because the client is extremely weak and can't produce an effective cough, the nurse should monitor closely for: A. Pleural effusion B. Pulmonary edema C. Atelectasis D. Oxygen toxicity

C. Atelectasis In a client with COPD, an ineffective cough impedes secretion removal. This, in turn, causes mucus plugging, which leads to localized airway obstruction — a known cause of atelectasis. Adults with COPD have extensive collateral ventilation secondary to airway destruction and thus are less likely to develop resorption atelectasis in the presence of an obstructing lesion (i.e., intrathoracic tumor). The use of high inspiratory oxygen concentration (high FiO2) during induction and maintenance of general anesthesia also contributes to atelectasis via absorption atelectasis.

Nurse Joy is caring for a client after a bronchoscopy and biopsy. Which of the following signs, if noticed in the client, should be reported immediately to the physician? A. Dry cough B. Hematuria C. Bronchospasm D. Blood-streaked sputum

C. Bronchospasm If a biopsy was performed during a bronchoscopy, blood-streaked sputum is expected for several hours. The client should be assessed for signs of complications, which would include cyanosis, dyspnea, stridor, bronchospasm, hemoptysis, hypotension, tachycardia, and dysrhythmias. Cardiac arrhythmias may also occur especially in patients with pre-existing cardiac disease.

Miriam, a college student with acute rhinitis sees the campus nurse because of excessive nasal drainage. The nurse asks the patient about the color of the drainage. In acute rhinitis, nasal drainage normally is: A. Yellow B. Green C. Clear D. Gray

C. Clear Normally, nasal drainage in acute rhinitis is clear. Anterior rhinoscopy typically reveals swelling of the nasal mucosa and thin, clear secretions. The inferior turbinates may take on a bluish hue, and cobblestoning of the nasal mucosa may be present. On physical examination, clinicians may notice mouth breathing, frequent sniffling and/or throat clearing, transverse supra-tip nasal crease, and dark circles under the eyes (allergic shiners). Option A: Yellow drainage indicates spread of the infection to the sinuses. Yellow mucus is a sign that whatever virus or infection the client has is taking hold. The body is fighting back. The yellow color comes from the cells — white blood cells, for example — rushing to kill the offending germs. Once the cells have done their work, they're discarded in the drainage and tinge it a yellowish-brown. Option B: Green drainage may also indicate infection. If the immune system kicks into high gear to fight infection, the drainage may turn green and become especially thick. The color comes from dead white blood cells and other waste products. Some sinus infections may be viral, not bacterial. Option D: Gray drainage may indicate a secondary infection. This could be a fungal sinus infection. These are different from viral or bacterial infections because the fungi feeds on the nasal tissue—and reproduces. Fungal sinus infections may occur due to a previous nasal injury or long-term nasal inflammation, as well as a weakened immune system. Growths called "fungus balls" develop in the cheek sinus as clumps of fungal spores. The fungus balls must be removed by surgery.

A client with acute asthma is prescribed short-term corticosteroid therapy. What is the rationale for the use of steroids in clients with asthma? A. Corticosteroids promote bronchodilation. B. Corticosteroids act as an expectorant. C. Corticosteroids have an anti-inflammatory effect. D. Corticosteroids prevent development of respiratory infections.

C. Corticosteroids have an anti-inflammatory effect Corticosteroids have an anti-inflammatory effect and act to decrease edema in the bronchial airways and decrease mucus secretion. At a physiologic level, steroids reduce airway inflammation and mucus production and potentiate beta-agonist activity in smooth muscles and reduce beta-agonists tachyphylaxis in patients with severe asthma. Corticosteroids do not have a bronchodilator effect, act as expectorants, or prevent respiratory infections.

A female client must take streptomycin for tuberculosis. Before therapy begins, the nurse should instruct the client to notify the physician if a health concern occurs? A. Impaired color discrimination B. Increased urinary frequency C. Decreased hearing acuity D. Increased appetite

C. Decreased hearing acuity Decreased hearing acuity indicates ototoxicity, a serious adverse effect of streptomycin therapy. The client should notify the physician immediately if it occurs so that streptomycin can be discontinued and an alternative drug can be prescribed. Ototoxicity and vestibular impairment are often thought to be the hallmark of streptomycin toxicity. In extreme cases, deafness may occur due to ototoxicity, thus caution must be exercised when combining streptomycin with other potentially ototoxic drugs. The other options aren't associated with streptomycin. Option A: Impaired color discrimination indicates color blindness. There are also reports of neuromuscular blockade with streptomycin use in association with installation into body cavities, use during anesthesia involving the use of neuromuscular blocking agents, and overdose in children. Neurotoxic effects can lead to optic nerve dysfunction, peripheral neuritis, and encephalopathy. Option B: Increased urinary frequency accompanies diabetes mellitus. Monitoring for streptomycin toxicity is especially important in the young and patients with renal impairment, as streptomycin occurs via glomerular filtration. Renal impairment can lead to a prolonged half-life of 50 to 100 hours. Option D: Increased appetite is not associated with streptomycin. Monitoring is based on the limited therapeutic index of aminoglycosides and known toxicities, particularly nephrotoxicity and ototoxicity. In general, clinicians should avoid concomitant use of additional medications with possible ototoxic or nephrotoxic effects.

After undergoing a left pneumonectomy, a female patient has a chest tube in place for drainage. When caring for this patient, the nurse must: A. Monitor fluctuations in the water-seal chamber. B. Clamp the chest tube once every shift. C. Encourage coughing and deep breathing. D. Milk the chest tube every 2 hours.

C. Encourage coughing and deep breathing When caring for a patient who is recovering from a pneumonectomy, the nurse should encourage coughing and deep breathing to prevent pneumonia in the unaffected lung. Assist the patient with splinting painful areas when coughing, deep breathing. Supporting chest and abdominal muscles makes coughing more effective and less traumatic. Option A: Because the lung has been removed, the water-seal chamber should display no fluctuations. Bubbling during expiration reflects venting of pneumothorax (desired action). Bubbling usually decreases as the lung expands or may occur only during expiration or coughing as the pleural space diminishes. The absence of bubbling may indicate complete lung re-expansion (normal) or represent complications such as an obstruction in the tube. Option B: Reinflation is not the purpose of a chest tube. Know the location of air leak (patient- or system-centered) by clamping thoracic catheter just distal to exit from the chest. If bubbling stops when the catheter is clamped at the insertion site, leak is patient-centered (at the insertion site or within the patient). Option D: Chest tube milking is controversial and should be done only to remove blood clots that obstruct the flow of drainage. Although routine stripping is not recommended, it may be necessary occasionally to maintain drainage in the presence of fresh bleeding, large blood clots, or purulent exudate (empyema).

A male patient's X-ray result reveals bilateral white-outs, indicating adult respiratory distress syndrome (ARDS). This syndrome results from: A. Cardiogenic pulmonary edema B. Respiratory alkalosis C. Increased pulmonary capillary permeability D. Renal failure

C. Increased pulmonary capillary permeability ARDS results from increased pulmonary capillary permeability, which leads to noncardiogenic pulmonary edema. ARDS is defined as an acute disorder that starts within 7 days of the inciting event and is characterized by bilateral lung infiltrates and severe progressive hypoxemia in the absence of any evidence of cardiogenic pulmonary edema. ARDS is defined by the patient's oxygen in arterial blood (PaO2) to the fraction of the oxygen in the inspired air (FiO2). These patients have a PaO2/FiO2 ratio of less than 300.

Immediately following a thoracentesis, which clinical manifestations indicate that a complication has occurred and the physician should be notified? A. Serosanguineous drainage from the puncture site. B. Increased temperature and blood pressure. C. Increased pulse and pallor. D. Hypotension and hypothermia.

C. Increased pulse and pallor Increased pulse and pallor are symptoms associated with shock. A compromised venous return may occur if there is a mediastinal shift as a result of excessive fluid removal. Usually, no more than 1 L of fluid is removed at one time to prevent this from occurring. Option A: Complications include bleeding, pain, and infection at the point of needle entry. If the approach is made too high in the intercostal space damage to the coastal vasculature and nerve injury is possible. Option B: If too much fluid is removed or if the fluid is removed too rapidly (eg using negative pressure chambers) re-expansion (aka post-expansion) pulmonary edema may occur. Removal of significant fluid volumes may also induce vasovagal physiology. Option D: If the procedural needle/catheter is passed through diseased tissue prior to entering the chest cavity, that process can be extended into the chest space. For example, passing the needle through thoracic or pleural tumor can seed the thoracic cavity or passing the needle through a chest wall abscess or otherwise infected tissue can result in empyema.

The most reliable index to determine the respiratory status of a client is to: A. Observe the chest rising and falling. B. Observe the skin and mucous membrane color. C. Listen and feel the air movement. D. Determine the presence of a femoral pulse.

C. Listen and feel the air movement To check for breathing, the nurse places her ear and cheek next to the client's mouth and nose to listen and feel for air movement. During the inspection, the examiner should pay attention to the pattern of breathing: thoracic breathing, thoracoabdominal breathing, coastal markings, and use of accessory breathing muscles. The use of accessory breathing muscles (i.e., scalenes, sternocleidomastoid muscle, intercostal muscles) could point to excessive breathing effort caused by pathologies.

A nurse is suctioning fluids from a female client through an endotracheal tube. During the suctioning procedure, the nurse notes on the monitor that the heart rate is decreasing. Which if the following is the appropriate nursing intervention? A. Continue to suction. B. Notify the physician immediately. C. Stop the procedure and reoxygenate the client. D. Ensure that the suction is limited to 15 seconds.

C. Stop the procedure and reoxygenate the client During suctioning, the nurse should monitor the client closely for side effects, including hypoxemia, cardiac irregularities such as a decrease in heart rate resulting from vagal stimulation, mucosal trauma, hypotension, and paroxysmal coughing. If side effects develop, especially cardiac irregularities, the procedure is stopped and the client is reoxygenated.

For a female patient with chronic obstructive pulmonary disease, which nursing intervention would help maintain a patent airway? A. Restricting fluid intake to 1,000 ml per day. B. Enforcing absolute bed rest. C. Teaching the patient how to perform controlled coughing. D. Administering prescribed sedatives regularly and in large amounts.

C. Teaching the patient how to perform controlled coughing

The nurse is teaching a male client with chronic bronchitis about breathing exercises. Which of the following should the nurse include in the teaching? A. Make inhalation longer than exhalation. B. Exhale through an open mouth. C. Use diaphragmatic breathing. D. Use chest breathing.

C. Use diaphragmatic breathing In chronic bronchitis the diaphragm is flat and weak. Diaphragmatic breathing helps to strengthen the diaphragm and maximizes ventilation. When the client has COPD, air often becomes trapped in the lungs, pushing down on the diaphragm. The neck and chest muscles must then assume an increased share of the work of breathing. This can leave the diaphragm weakened and flattened, causing it to work less efficiently. Option A: Exhalation should be longer than inhalation to prevent collapse of the bronchioles. Never allow a patient to force expiration. expiration should be relaxed or lightly controlled. forced expiration only increases turbulence in the airways leading to bronchospasm and increased airway restriction. Option B: The client with chronic bronchitis should exhale through pursed lips to prolong exhalation, keep the bronchioles from collapsing, and prevent air trapping. The client should tighten his stomach muscles, letting them fall inward as he exhales through pursed lips. The hand on his upper chest must remain as still as possible. Option D: Diaphragmatic breathing — not chest breathing — increases lung expansion. Controlled breathing techniques, which emphasize diaphragmatic breathing are designed to improve the efficiency of ventilation, decrease the work of breathing, increase the excursion of the diaphragm, and improve gas exchange and oxygenation.

For a patient with advanced chronic obstructive pulmonary disease (COPD), which nursing action best promotes adequate gas exchange? A. Encouraging the patient to drink three glasses of fluid daily. B. Keeping the patient in semi-Fowler's position. C. Using a high-flow venturi mask to deliver oxygen as prescribed. D. Administering a sedative, as prescribed.

C. Using a high-slow venturi mask to deliver oxygen as prescribed The patient with COPD retains carbon dioxide, which inhibits stimulation of breathing by the medullary center in the brain. As a result, low oxygen levels in the blood stimulate respiration, and administering unspecified, unmonitored amounts of oxygen may depress ventilation. To promote adequate gas exchange, the nurse should use a Venturi mask to deliver a specified, controlled amount of oxygen consistently and accurately. Option A: Drinking three glasses of fluid daily would not affect gas exchange or be sufficient to liquefy secretions, which are common in COPD. Increase fluid intake to 3000 mL per day within cardiac tolerance. Provide warm or tepid liquids. Recommend the intake of fluids between, instead of during, meals. Hydration helps decrease the viscosity of secretions, facilitating expectoration. Option B: Patients with COPD and respiratory distress should be placed in high-Fowler's position. Elevation of the head of the bed facilitates respiratory function by use of gravity; however, the patient in severe distress will seek the position that most eases breathing. Supporting arms and legs with table, pillows, and so on helps reduce muscle fatigue and can aid chest expansion. Option D: They should not receive sedatives or other drugs that may further depress the respiratory center. Assess the patient's respiratory response to activity which includes monitoring of respiratory rate and depth, oxygen saturation, and use of accessory muscles for respiration. Patients with COPD can experience hypoxia during increased activity and may need oxygenation to avoid hypoxemia which puts them at risk for exacerbations of the condition.

The nurse teaches a client with COPD to assess for s/s of right-sided heart failure. Which of the following s/s would be included in the teaching plan? A. clubbing of nail beds B. hypertension C. peripheral edema D. increased appetite

C. peripheral edema Right-sided heart failure is a complication of COPD that occurs because of pulmonary hypertension. Signs and symptoms of right-sided heart failure include peripheral edema, jugular venous distention, hepatomegaly, and weight gain due to increased fluid volume. Right heart failure is most commonly a result of left ventricular failure via volume and pressure overload. Clinically, patients will present with signs and symptoms of chest discomfort, breathlessness, palpitations, and body swelling.

Nurse Oliver observes constant bubbling in the water-seal chamber of a closed chest drainage system. What should the nurse conclude? A. The system is functioning normally. B. The client has a pneumothorax. C. The system has an air leak. D. The chest tube is obstructed.

C. the system has a air leak Constant bubbling in the chamber indicates an air leak and requires immediate intervention. With suction applied, this indicates a persistent air leak that may be from a large pneumothorax at the chest insertion site (patient-centered) or chest drainage unit (system-centered). Option A: Clients without a pneumothorax should have no evidence of bubbling in the chamber. Absence of bubbling may indicate complete lung re-expansion (normal) or represent complications such as obstruction in the tube. Option B: The client with a pneumothorax will have intermittent bubbling in the water-seal chamber. Bubbling during expiration reflects venting of pneumothorax (desired action). Bubbling usually decreases as the lung expands or may occur only during expiration or coughing as the pleural space diminishes. Option D: If the tube is obstructed, the nurse should notice that the fluid has stopped fluctuating in the water-seal chamber. Monitor water-seal chamber "tidaling." Note whether the change is transient or permanent. Tidaling of 2-6 cm during inspiration is normal and may increase briefly during coughing episodes. Continuation of excessive tidal fluctuations may indicate the existence of airway obstruction or the presence of a large pneumothorax.

Blessy, a community health nurse is conducting an educational session with community members regarding tuberculosis. The nurse tells the group that one of the first symptoms associated with tuberculosis is: A. Dyspnea B. Chest pain C. A bloody, productive cough. D. A cough with the expectoration of mucoid sputum.

D. A cough with the expectoration of mucoid sputum One of the first pulmonary symptoms is a slight cough with the expectoration of mucoid sputum. A chronic cough, hemoptysis, weight loss, low-grade fever, and night sweats are some of the most common physical findings in pulmonary tuberculosis. Other options are late symptoms and signify cavitation and extensive lung involvement. Option A: Secondary tuberculosis differs in clinical presentation from the primary progressive disease. In secondary disease, the tissue reaction and hypersensitivity is more severe, and patients usually form cavities in the upper portion of the lungs Option B: Pulmonary or systemic dissemination of the tubercles may be seen in active disease, and this may manifest as miliary tuberculosis characterized by millet-shaped lesions on chest x-ray. Disseminated tuberculosis may also be seen in the spine, the central nervous system, or the bowel. Option C: As the bacterium begins multiplying in the body and destroying tissue, it causes symptoms such as a bad, persistent cough, fatigue/loss of energy, weight loss, loss of appetite, chills, fever, drenching night sweats, chest pain, and coughing up or spitting up bright red blood, a symptom that occurs when the blood vessels inside the lungs become eroded and begin to bleed.

Which of the following outcomes would be appropriate for a client with COPD who has been discharged to home? The client: A. Promises to do PLB at home B. States actions to reduce pain C. States that he will use oxygen via a nasal cannula at 5 L/min D. Agrees to call the physician if dyspnea on exertion increases

D. Agrees to call the physician if dyspnea on exertion increases. Increasing dyspnea on exertion indicates that the client may be experiencing complications of COPD, and therefore the physician should be notified. There are things that everyone with COPD should do to manage their disease; quitting smoking (if they smoke) is the most important. In addition, there are other non-medication treatments that can help relieve symptoms and improve quality of life.

A female client is undergoing a complete physical examination as a requirement for college. When checking the client's respiratory status, the nurse observes respiratory excursion to help assess: A. Lung vibrations B. Vocal sounds C. Breath sounds D. Chest movements.

D. Chest movements The nurse observes respiratory excursion to help assess chest movements. Normally, thoracic expansion is symmetrical; unequal expansion may indicate pleural effusion, atelectasis, pulmonary embolism, or a rib or sternum fracture. During the inspection, the examiner should pay attention to the pattern of breathing: thoracic breathing, thoracoabdominal breathing, coastal markings, and use of accessory breathing muscles. The use of accessory breathing muscles (i.e., scalenes, sternocleidomastoid muscle, intercostal muscles) could point to excessive breathing effort caused by pathologies. Option A: After asking the client to say "99," the nurse palpates the vibrations transmitted from the bronchopulmonary system along the solid surfaces of the chest wall to the nurse's palms. An increase in the tactile fremitus points towards an increased intraparenchymal density and a decreased fremitus hints towards a pleural process that separates the pleura from the parenchyma (pleural effusion, pneumothorax). Option B: The nurse assesses vocal sounds to evaluate air flow when checking for tactile fremitus. Palpation should focus on detecting abnormalities like masses or bony crepitus. Of note, the fremitus can also be auscultated and can be referred to as vocal fremitus. Option C: The nurse assesses breath sounds during auscultation. The movement of air generates normal breath sounds through the large and small airways. Normal breath sounds have a frequency of approximately 100 Hz. The absence of breath sounds should prompt the health care provider to consider shallow breath, abnormal anatomy, or pathologic entities such as airway obstruction, bulla, hyperinflation, pneumothorax, pleural effusion or thickening, and obesity.

Nurse Ruth assessing a patient for tracheal displacement should know that the trachea will deviate toward the: A. Contralateral side in a simple pneumothorax. B. Affected side in a hemothorax. C. Affected side in a tension pneumothorax. D. Contralateral side in hemothorax.

D. Contralateral side on hemothorax The trachea will shift according to the pressure gradients within the thoracic cavity. If there is no significant air or fluid accumulation, the trachea will not shift. The pressure gradient inside the thorax changes with a pneumothorax. Normally the pressure of the pleural space is negative when compared to atmospheric pressure. When the chest wall expands outwards, the lung also expands outwards due to surface tension between parietal and visceral pleura.

Before administering ephedrine, Nurse Tony assesses the patient's history. Because of ephedrine's central nervous system (CNS) effects, it is not recommended for: A. Patients with an acute asthma attack. B. Patients with narcolepsy. C. Patients under age D. Elderly patients.

D. Elderly patients Ephedrine is not recommended for elderly patients, who are particularly susceptible to CNS reactions (such as confusion and anxiety) and to cardiovascular reactions (such as increased systolic blood pressure, coldness in the extremities, and anginal pain). Ephedrine is also arrhythmogenic, and caution should be used during administration to patients who are predisposed to arrhythmias or taking other arrhythmogenic medications, particularly digitalis.

A male adult patient on mechanical ventilation is receiving pancuronium bromide (Pavulon), 0.01 mg/kg I.V. as needed. Which assessment finding indicates that the patient needs another pancuronium dose? A. Leg movement B. Finger movement C. Lip movement D. Fighting the ventilator

D. Fighting the ventilator Pancuronium, a non-depolarizing blocking agent, is used for muscle relaxation and paralysis. It assists mechanical ventilation by promoting endotracheal intubation and paralyzing the patient so that the mechanical ventilator can do its work. Fighting the ventilator is a sign that the patient needs another pancuronium dose. The nurse should administer 0.01 to 0.02 mg/kg I.V. every 20 to 60 minutes. Movement of the legs, or lips has no effect on the ventilator and therefore is not used to determine the need for another dose.

Which of the following diets would be most appropriate for a client with COPD? A. Low fat, low cholesterol B. Bland, soft diet C. Low-sodium diet D. High calorie, high protein diet

D. High calorie, low fat diet

A male patient is admitted to the healthcare facility for treatment of chronic obstructive pulmonary disease. Which nursing diagnosis is most important for this patient? A. Activity intolerance related to fatigue. B. Anxiety related to actual threat to health status. C. Risk for infection related to retained secretions. D. Impaired gas exchange related to airflow obstruction.

D. Impaired gas exchange related to airflow obstruction A patient airway and an adequate breathing pattern are the top priority for any patient, making "impaired gas exchange related to airflow obstruction" the most important nursing diagnosis. Monitor O2 saturation and titrate oxygen to maintain Sp02 between 88% to 92%. Pulse oximetry reading of 87% below may indicate the need for oxygen administration while a pulse oximetry reading of 92% or higher may require oxygen titration. The other options also may apply to this patient but less important. Option A: Patients with COPD experience progressive activity and exercise intolerance. Evaluation of the patient's activity tolerance and limitations helps create strategies to promote independent ADLs. Assess the patient's respiratory response to activity which includes monitoring of respiratory rate and depth, oxygen saturation, and use of accessory muscles for respiration. Option B: Ineffective Coping may be related to decreased socialization, depression, anxiety, and inability to work. Provide instructions for self-management of COPD. Assessment of the patient's knowledge and including family members about the therapeutic regimen increases adherence to treatment regimen. Option C: Respiratory infections that are minor in nature may be threatening to people with COPD. Bronchopulmonary infections must be controlled or prevented to diminish inflammatory edema. Review the importance of breathing exercises, effective cough, frequent position changes, and adequate fluid intake.

A nurse is assisting a physician with the removal of a chest tube. The nurse should instruct the client to: A. Exhale slowly B. Stay very still C. Inhale and exhale quickly D. Perform the Valsalva maneuver

D. Perform the Valsalva maneuver When the chest tube is removed, the client is asked to perform the Valsalva maneuver (take a deep breath, exhale, and bear down). The tube is quickly withdrawn, and an airtight dressing is taped in place. An alternative instruction is to ask the client to take a deep breath and hold the breath while the tube is removed.

Pseudoephedrine (Sudafed) has been ordered as a nasal decongestant. Which of the following is a possible side effect of this drug? A. constipation B. bradycardia C. diplopia D. restlessness

D. Restlessness Side effects of pseudoephedrine are experienced primarily in the cardiovascular system and through sympathetic effects on the CNS. The most common CNS effects include restlessness, dizziness, tension, anxiety, insomnia, and weakness. Common cardiovascular side effects include tachycardia, hypertension, palpitations, and arrhythmias.

Which of the following is the primary reason to teach pursed-lip breathing to clients with emphysema? A. To promote oxygen intake B. To strengthen the diaphragm C. To strengthen the intercostal muscles D. To promote carbon dioxide elimination

D. To promote carbon dioxide elimination Pursed lip breathing prolongs exhalation and prevents air trapping in the alveoli, thereby promoting carbon dioxide elimination. By prolonged exhalation and helping the client relax, pursed-lip breathing helps the client learn to control the rate and depth of respiration. Pursed-lip breathing does not promote the intake of oxygen, strengthen the diaphragm, or strengthen intercostal muscles.

The best method of oxygen administration for client with COPD uses: A. Cannula B. Simple Face mask C. Non-rebreather mask D. Venturi mask

D. Venturi Mask Venturi delivers controlled oxygen. An air-entrainment (also known as venturi) mask can provide a pre-set oxygen to the patient using jet mixing. As the percent of inspired oxygen increases using such a mask, the air-to-oxygen ratio decreases, causing the maximum concentration of oxygen provided by an air-entrainment mask to be around 40%. Option A: A thin tube, often affixed behind the ears and used to deliver oxygen directly to the nostrils from a source connected with tubing. This is the most common method of delivery for home use and provides flow rates of 2 to 6 liters per minute (LPM) comfortably, allowing the delivery of oxygen while maintaining the patient's ability to utilize his or her mouth to talk, eat, etc. Option B: Facemasks can be generally divided into simple face masks, air-entrainment masks, and non-rebreathers. A simple facemask is a mask with no bag attached, which delivers oxygen at 5 to 8 LPM. A disadvantage of this and other full face masks is the inability of the patient to eat, drink, or easily communicate while using such a device. Option C: Non-rebreathing masks have a bag attached to the mask known as a reservoir bag, which inhalation draws from to fill the mask through a one-way valve and features ports at each side for exhalation, resulting in an ability to provide the patient with 100% oxygen at a higher LPM flow rate.

A 34-year-old woman with a history of asthma is admitted to the emergency department. The nurse notes that the client is dyspneic, with a respiratory rate of 35 breaths/minute, nasal flaring, and use of accessory muscles. Auscultation of the lung fields reveals greatly diminished breath sounds. Based on these findings, what action should the nurse take to initiate care of the client? A. initiate oxygen therapy and reassess the client in 10 minutes B. draw blood for an ABG and send the client for a CXR C. encourage the client to relax and breathe slowly through the mouth D. administer bronchodilators

D. administer bronchodilators In an acute asthma attack, diminished or absent breath sounds can be an ominous sign indicating lack of air movement in the lungs and impending respiratory failure. The client requires immediate intervention with inhaled bronchodilators, intravenous corticosteroids, and possibly intravenous theophylline.

The nurse assesses the respiratory status of a client who is experiencing an exacerbation of COPD secondary to an upper respiratory tract infection. Which of the following findings would be expected? A. normal breath sounds B. prolonged inspiration C. normal chest movement D. coarse crackles and rhonchi

D. coarse crackles and rhonchi Exacerbations of COPD are frequently caused by respiratory infections. Coarse crackles and rhonchi would be auscultated as air moves through airways obstructed with secretions. Crackles are usually due to airway secretions within a large airway and disappear on coughing. These crackles are scanty, gravity-independent, usually audible at the mouth, and strongly associated with severe airway obstruction. Option A: In COPD, breath sounds are diminished because of an enlarged anteroposterior diameter of the chest. A reduction in breath sound intensity (BSI) is often seen in patients with COPD. Pardee et al. developed a scoring system for BSI. According to this system, the clinician listens sequentially over six locations on the patient's chest: bilaterally over the upper anterior portion of the chest, in the midaxillary, and at the posterior bases. Option B: Expiration, not inspiration, becomes prolonged. Patients with COPD often present with diminished breath sounds, prolonged expiratory time, and expiratory wheezing that initially may occur only on forced expiration. Option C: Chest movement is decreased as lungs become overdistended. Additional findings on physical examination include hyperinflation of the lungs with an increased anteroposterior chest diameter ("barrel chest"); use of accessory muscles of respiration; and distant heart sounds, sometimes best heard in the epigastrium.

A nurse instructs a female client to use the pursed-lip method of breathing and the client asks the nurse about the purpose of this type of breathing. The nurse responds, knowing that the primary purpose of pursed-lip breathing is to: A. Promote oxygen intake. B. Strengthen the diaphragm. C. Strengthen the intercostal muscles. D. Promote carbon dioxide elimination.

D. promote CO2 elimination Pursed-lip breathing facilitates maximal expiration for clients with obstructive lung disease. This type of breathing allows better expiration by increasing airway pressure that keeps air passages open during exhalation. Pursed-lip breathing is a technique that allows people to control their oxygenation and ventilation. The technique requires a person to inspire through the nose and exhale through the mouth at a slow controlled flow.

What is a face tent used for?

Face tents are used to provide a controlled concentration of oxygen and increase moisture for patients who have facial burn or a broken nose, or who are claustrophobic. The mask covers the nose and mouth and does not create a seal around the nose. It can provide 28% to 100% O2 Flow meter should be set to deliver O2 at a minimum of 15 L/min. It is difficult to achieve high levels of oxygenation with this mask.

What is the tracheostomy collar used for?

One is to use a tracheostomy collar, which is placed over a breathing tube in a tracheotomy incision in the throat, and through which humidified oxygen is given. The other is to reduce the pressure support supplied via the ventilator. A study found tracheostomy collars significantly outperformed pressure support in helping patients breathe on their own again.

Which phrase is used to describe the volume of air inspired and expired with a normal breath? A. Total lung capacity B. Forced vital capacity C. Tidal volume D. Residual volume

Tidal Volume

Bronchoscopy =

a procedure to look directly at the airways in the lungs using a thin, lighted tube (bronchoscope). The bronchoscope is put in the nose or mouth. It is moved down the throat and windpipe (trachea), and into the airways.

What is a pneumothorax?

collapsed lung caused by air leaking into the space between your lung and chest wall. Pushes on the outside of your lung making it collapse.

Bronchospasm =

occurs when the airways (bronchial tubes) go into spasm and contract. This makes it hard to breathe and causes wheezing

pallor = cyanosis =

pale blue

Residual Volume

the maximal amount of air left in the lung after a maximal expiration. In other words, it is the volume of air that cannot be expelled, thus causing the alveoli to remain open at all times. The residual volume remains unchanged regardless of the lung volume at which expiration was started.

Total Lung Capacity

the maximal amount of air the lungs and respiratory passages can hold after a forced inspiration. Among healthy adults, the average lung capacity is about 6 liters.

Forced vital capacity

the vital capacity performed with a maximally forced expiration


Conjuntos de estudio relacionados

NUR 113 - MS PrepU Practice Questions - Patel Test 4

View Set

Unit - 4 Mechanisms of Evolution

View Set

Chapter 15 - The Lymphatic System

View Set

(Practice) Ch. 14 - Escrow and Title Insurance

View Set

Introduction to Exercise & Wellness Mid-Term Exam

View Set